It is fully determined which grants are awarded for each quarter of a particular calendar year if which one of the fo...

farnoushsalimian on October 7, 2022

W in quarter 4

Confused because three grants in quarter three allows for both W's inclusion and W's exclusion from quarter four.... we don't know if that grant is given or not. How can the set be said to be fully determined?

Reply
Create a free account to read and take part in forum discussions.

Already have an account? log in

Emil-Kunkin on October 19, 2022

We do know that W must be in 4. We must have 6 total, and one grant in each quarter, and W is the only thing that can go in 4, so it must go in 4.